4x + 7y <14

Into slope intercept form

Slope
Y-intercept

Answers

Answer 1

Answer: y < [tex]\frac{4}{7}[/tex]x + 2

Slope: [tex]\frac{4}{7}[/tex]

Y-intercept: 2


Related Questions


[tex]( {2}^{ - 1} + {3}^{ - 1} )^{2} [/tex]
solve.

will give the brainliest​

Answers

Answer:

25/36

Step-by-step explanation:

(2^-1 + 3^-1)^2

(1/2 + 1/3)^2

(5/6)^2 = 25/36

25/36 or .694

2^-1= 1/2
3^-1=1/3

Add the together
5/6 square it and you get 25/36

Which function is represented by the graph below?

Answers

Answer:

y= 2x-4

Step-by-step explanation:

The y-intercept is at -4, it is a positive slope, plotting from the y-axis rise over run is equal to 2/1 which means that the slope is 2x. giving you y= 2x-4

The equation which satisfy to the graph is Y= 2x- 4

What is an equation?

An equation is a mathematical expression that contains an equals symbol. Equations often contain algebra.

According to graph

There are multiples points given on which line was passing

But we will select only which is clear

1st point = (0, 2)

2nd point = (-4,0 )

Now we will put values on equations and check which will satisfy

For 1st equation

Y= 2x- 4

Checking with 1st point

Putting x= 2

Y= 0 which satisfy

Checking for 2nd point

Putting x=0

Y = -4 satisfy

Hence, equation which satisfy to graph is Y= 2x- 4

To know more about equation here :

https://brainly.com/question/24169758

# SPJ2

PLEASE HELP WILL GIVE BRAINLIEST AND 20 POINTS

solve this system of equations

3y-5x=12
y=1/3x

Answers

Answer:

x=-3

y=-1

(-3,-1)

Answer:

(-3, -1).

Step-by-step explanation:

I have attached the work to your problem.

Please see the attachment below.

I hope this helps!

Is the relation a function?
{(-6, -1), (5,-1), (0, -1), (-2, -1), (3, -1)}

Answers

Answer:

yes it is a function

Step-by-step explanation:

For that no two x values are the same with different y values

Line I and h intersect at what point

Answers

Answer:

please show where the lines are graphed in a picture

Step-by-step explanation:

evaluate the following expression -7x(7+9)

Answers

Answer:

-112

Step-by-step explanation:

Add

-7(7+9)

Multiply

-7x16

Wallah! You have the answer

-112

[tex]\text{Simplify the expression:}\\\\-7x(7+9)\\\\\text{Use the distributive property}\\\\-49x-63x\\\\\text{Combine like terms}\\\\\boxed{-112x}[/tex]

14. Roger is on a playground swing, and he is swinging back and forth in such a way that the height, h, in feet, of the swing off the ground is given by the equation h=3cos(3π/2t) +5, where t is in seconds. How many seconds elapses between two consecutive times that the swing is at its maximum height?​

Answers

Answer:

The time [tex]t = \frac{3}{2}[/tex]  seconds elapses between two consecutive times that the swing is at its maximum height  'h' = 2

 Step-by-step explanation:

Explanation:-

Step(i):-

Given function [tex]h(t) = 3 cos (\frac{3\pi }{2 t} ) +5[/tex] ....(i)

By using derivative formulas

[tex]\frac{d cosx }{d x} = -sinx[/tex]

[tex]\frac{d x^{n} }{d x} = n x^{n-1}[/tex]

[tex]\frac{d t^{-1} }{d x} = -1 t^{-1-1} = - t^{-2} = \frac{-1}{t^{2} }[/tex]

Step(ii):-

Differentiating equation(i) with respective to 't'

[tex]h^{l} (t) = 3(-sin(\frac{3\pi }{2t})\frac{d}{dt} (\frac{3\pi }{2t } )+0[/tex]  ...(ii)

[tex]h^{l} (t) = 3(-sin(\frac{3\pi }{2t})(\frac{-3\pi }{2t^{2} } )+0[/tex]

Equating zero

[tex]h^{l} (t) = 3(-sin(\frac{3\pi }{2t})(\frac{-3\pi }{2t^{2} } )=0[/tex]

[tex]3(-sin(\frac{3\pi }{2t})(\frac{-3\pi }{2t^{2} } ) = 0[/tex]

on simplification , we get

[tex](sin(\frac{3\pi }{2t}) = 0[/tex]

now we use formulas

sin 0 = 0 and sinπ = 0

General solution

[tex](sin(\frac{3\pi }{2t}) = sin\pi[/tex]

[tex](\frac{3\pi }{2t}) = \pi[/tex]

Cancellation 'π' on both sides, we get

[tex]3 = 2 t[/tex]

Dividing '2' on both sides , we get

[tex]t = \frac{3}{2}[/tex]

Again differentiating with respective to 't' , we get  

[tex]h^{ll} (t) = 3(-cos(\frac{3\pi }{2t})(\frac{-3\pi }{2t^{2} } )+ (-3)(-sin(\frac{3\pi }{2t} )(\frac{6\pi }{2t^{3} }[/tex]

Put t= 3/2 and simplification

[tex]h^{ll} (t) < 0[/tex]

The maximum height

                   [tex]h(t) = 3 cos (\frac{3\pi }{2 t} ) +5[/tex]

               [tex]h(\frac{3}{2} ) = 3 cos (\frac{3\pi }{2(\frac{3}{2} )} )+5[/tex]

              [tex]h(\frac{3}{2} ) = 3 cos (\pi )+5 = -3+5 =2[/tex]

[tex]t = \frac{3}{2}[/tex]  seconds elapses between two consecutive times that the swing is at its maximum height  'h' = 2

Conclusion:-

The time [tex]t = \frac{3}{2}[/tex]  seconds elapses between two consecutive times that the swing is at its maximum height  'h' = 2

 

 

HELP ASAP it says my questions should be at least 20 characters long so what I’m saying right now doesn’t mean anything.

Answers

Answer: See pic above for answer. I got it from Photomath

Plz put brainliest

Solve:
-3(7p + 5) = 27
Helpppp

Answers

Answer:

-2

Step-by-step explanation:

-21p - 15 = 27

-21p = 42

p = -2

Answer:

-2

Step-by-step explanation:

-21p - 15 = 27

-21p = 42

p = -2

I need help with is question ASAP please

Answers

The y-intercept looks like it's 0 and the slop looks like 6/2 or 3 or maybe 7/2

Tiffany cells 2 kinds of homemade tomato sauce

Answers

Answer:

b

Step-by-step explanation:

Let x represent the number of quarts of Tuscan sauce and

y represents the number of quarts of marinara sauce Tiffany makes.

A quart of Tuscan sauce requires 6 tomatoes and 1 cup of oil

x quarts requires 6x tomatoes and 1x cups of oil

A quart of her marinara sauce requires 5 tomatoes and 1.25 cups of oil

y quarts requires 5y tomatoes and 1.25 y cups of oil

She has 45 tomatoes and 10 cups of oil on hand.

So the constraints are

6x+5y≤45

1x+1.25y≤10

x>=0 and y>=0

After drawing a diagram of what's required to make 1 quart of Tuscan and 1 quart of Marinara sauce, you can more clearly choose answer choice (B) 6x + 5y < (or equal to) 45, x + 5/4y < (or equal to) 10, x>(or equal to)0, y>(or equal to)0.

please mark brainliest :)

the answer is B !!! hope it’s correct

The area of the base of the cone is 64π square millimeters, and the area of the lateral surface is 112π square millimeters. Find the radius r and slant height ℓ of the cone.

Answers

Answer:

Radius = 8 mm

Slant Height = 14 mm

Step-by-step explanation:

Base of a cone is a circle.

Area of circle is given as [tex]A=\pi r^{2}[/tex]

Where [tex]r[/tex] is the radius.

Given that

[tex]A = 64 \pi\ mm^2\\\Rightarrow \pi r^{2} = 64\pi \\\Rightarrow r^{2} = 64\\\Rightarrow r = 8\ mm[/tex]

Hence, radius is 8 mm.

Formula for Lateral Surface Area of a cone:

[tex]LSA = \pi rl[/tex]

Where [tex]r[/tex] is the radius and

[tex]l[/tex] is the slant height of cone

Given that [tex]LSA = 112 \pi\ mm^2[/tex]

[tex]\Rightarrow 112\pi = \pi \times 8 \times l\\\Rightarrow 8 \times l = 112\\\Rightarrow l = 14\ mm[/tex]

Hence, slant height is 14 mm

Soledad has a storage box. The box is 6 1/2 inches long, 4 3/4 inches wide and 7 inches tall. She wants to run a border around the top of the box. How much border does she need?

Answers

Answer:

She need 10.5 inches border

Step-by-step explanation:

Length of box =[tex]\frac{1}{2} inches[/tex]

Breadth of box =[tex]4\frac{3}{4} inches[/tex]

Breadth of box =[tex]\frac{19}{4} inches[/tex]

Height of box = 7 inches

Now we are given that Soledad wants to run a border around the top of the box.

So, Length of top = Perimeter = [tex]2(l+b)=2(\frac{1}{2}+\frac{19}{4})=10.5[/tex]

Hence She need 10.5 inches border

BEST ANSWER GETS TO CHOOSE BRAINLESST OR FOLLOW!​

Answers

Answer:13 is>

14 is =

15 is >

16 is<

17 is<

18 is =

Step-by-step explanation:

I just know plz mark brainliest

Becky made 4 quarts of chicken noodle soup in a big pot, then served 112-cup bowls of soup to 6 people. How much soup is left in the pot?

Answers

Answer:

1qt 3c

Step-by-step explanation:



A new car is purchased for 23400 dollars. The value of the car depreciates at 11.5% per year. To the nearest year, how long will it be until the value of the car is 12700 dollars?

Answers

Answer: 5 years

Step-by-step explanation:

Using same formula as for Compound Interest:

12700 = 23400 (1-.115)^t

12700 = 23400 x 0.885^t

0.885^t = 127/234

Convert decimal into fraction:

177/200^t = 127/234

Take logarithm of both sides of thr equation:

t = log 177/200 (127/234)

t = 5.00242

Arrange from smallest to largest
0.01, 1.01, 10.01, 1.001

Answers

Answer:

0.01 --> 1.001 ---> 1.01 ---> 10.01

The numbers in the ascending order:

0.01, 1.01, 1.001, 10.01.

What is ascending order?

Putting numbers in ascending order simply means to do it from smallest to largest.

Given:

0.01, 1.01, 10.01, 1.001.

The numbers from smallest to largest:

That means, in the ascending order.

So,

0.01, 1.01, 1.001, 10.01.

Therefore, 0.01, 1.01, 1.001, 10.01 are in the required order.

To learn more about the ascending order;

https://brainly.com/question/20681445

#SPJ2

Find the solution(s) to x^2- 14x + 49 = 0.
O A. x=-2 and x = 7
B. x= -1 and x = 14
C. x= 7 only
D. x = 7 and x = -7

Answers

Answer:

[tex] \boxed{C. \: x = 7 \: only} [/tex]

Step-by-step explanation:

[tex] = > {x}^{2} - 14x + 49 = 0 \\ \\ = > {x}^{2} - (7 + 7)x + 49 = 0 \\ \\ = > {x}^{2} - 7x - 7x + 49 = 0 \\ \\ = > x(x - 7) - 7(x - 7) = 0 \\ \\ = > (x - 7)(x - 7) = 0 \\ \\ = > {(x - 7)}^{2} = 0 \\ \\ = > x - 7 = 0 \\ \\ = > x = 7[/tex]

What is the equation in slope-intercept form of a line with slope of 5 and y-intercept of 3?

Answers

y=5x+3

Because the starting value(y intercept is 3) and the slope(x) is 5

A. 256
B. 265
C. 297
D. 279​

Answers

Answer:

A.

Step-by-step explanation:

So calculate one of the triangles.

[tex]\frac{12*8}{2} =48[/tex]

48 * 4 = 192

192 + 64 = 256

Helppppp meeeee answer thissssssss

Answers

Answer:

(-1.025)^3 x (-1.025)^2=(- 1.025)^5

(-y)^3 x (-y)^2 x (-y)= y^6

5= odd, - stays

6= even, - becomes +

Answer:

5) (-1.025)^3* (-1.025)^2 = (-1.025)^5 = (-41/40)^5

10) -y^3 * -y^2 * -y

Multiplying an odd number of negative terms makes the product negative

= - y^3 * y^2 * y

= - y^6

If f(x) = 2x + 1 and g(x) = x-2 what is the value of f(g(f(3)))? This is so confusing plz help me will mark brainliest A) 1 B) 3 C) 5 D) 7 E) 11

Answers

Answer:

11

Step-by-step explanation:

f(3) = 2*3+1 = 6+1 = 7

Then find g(7)

g(7) = 7-2 = 5

Then find f(5)

f(5) = 2*5 +1 = 10+1 = 11

f(g(f(3))) = 11

Answer:

11

Step-by-step explanation:

f(x)=2x + 1

f(3)= 2.3 + 1

f(3)=7

f(g(f(3))) = f(g(7))

Now, we have to find g(7)

g(x)=x -2

g(7)= 7 -2

g(7)=5

f(g(f(3))) = f(g(7)) = f(5)

now we have to find f(5)

f(x)=2x + 1

f(5)=2.5 + 1

f(5)=11

Hope this helps ^-^

What is the midpoint of the segment below? (2,3)(-3,-2)

Answers

Answer:

(-0.5, 0.5)

Step-by-step explanation:

If those two points are endpoints, just use the midpoint formula.

(-0.5, 0.5)

Answer:

(-1/2,1/2)

Step-by-step explanation:

To find the midpoint, add the x coordinates and divide by 2

(2+-3)/2 = -1/2

add the y coordinates and divide by 2

(3+-2)/2 = 1/2

The midpoint is (-1/2,1/2)

please help as soon as possible
I WILL MARK YOU AS BRAINLIEST ​

Answers

Answer:

Last choice

Step-by-step explanation: Subtract 11 and then divide by 3. A would be greater than -2 and last choice shows that

00:00
Brice is finding the sum of 468 and 241 by breaking it into smaller problems
He uses place value and finds the sums of the hundreds, tens, and ones.
What is the sum of the tens? Enter your answer in the box.
1​

Answers

468+241=709 so 0 is in the tenth place because 6+4= 10 and one gets carried over to the 4+2

soooo i need help can yall find the answer for me I put mad points so better get it right .

Answers

Answer:

1.08 * 10 ^8

Step-by-step explanation:

2.4 * 10^4   * 4.5 * 10^3

Multiply the constants

10.8

Add the exponents

10^(4+3) = 10^7

Put together

10.8 * 10 ^7

Put we can only have one number in front of the decimal

Move the decimal one place to the left and add one to the exponent

1.08 * 10 ^8

plz help i will give brainliest!!! 2.5(4x+10)−2(3x−15)

Answers

Answer:

4x+55

Step-by-step explanation:

Expand and simplify

10x+25x-6x+30

4x+55

Answer:

4x+55

Step-by-step explanation:

I used the PEDMAS to simplify the equation doing the parentheses first then addition and subtraction. The picture attached showed what order I did it in and I hope this really helped! Good LUCK on the rest of your homework.

Which equation represents a line parallel to the line whose equations is -2x + 3y =
-4 and passes through the point (1,3)?

Answers

Answer:

2.  y - 3 = 2/3 (x - 1).

Step-by-step explanation:

-2x + 3y = -4

3y = 2x - 4

y = 2/3 x - 4/3 - so the slope is  2/3.

The slope of a line parallel to it is also 2/3.

It also passes through the point (1, 3).

Using the point-slope form of  a line:

y - y1 = m(x - x1) where m = the slope and (x1, y1) is a point on the line, we have:

y - 3 = 2/3 (x - 1)  <--- is the required equation.

Answer:

2

Step-by-step explanation:

parallel lines have same slope with different intercept

y= mx+b

m is going to be same with different b

so the given function is:

-2x+3y=-43y= 2x-4y= 2/3x - 4/3

Given options:

1. y-3= - 2/3(x-1)  ⇒ y= -2/3x +3 + 2/3 ⇒ y= -2/3x +11/3

it has different slope, so is not parallel

2. y-3= 2/3(x-1) ⇒ y= 2/3x+3-2/3 ⇒ y= 2/3x + 7/3

it has same slope, so is parallelit should be passing through point (1,3)3= 2/3+7/3 ⇒ 3=3, yes it does

3. y-3= -2/3(x+1) ⇒ y= - 2/3x +3- 2/3 ⇒ y= -2/3x + 7/3

it has different slope, is not parallel

4. y-3= 2/3(x+1) ⇒ y= 2/3x +3+ 2/3 ⇒ y= 2/3x +11/3

it has same slope, so is parallelit should be passing through point (1,3)3= 2/3+11/3 ⇒ 3≠13/3, no it doesn't

What is -6 divided by -1.2?

Answers

Answer:

5.

Step-by-step explanation:

This is the same as saying:

-6/(-6/5).

To solve the equation, we multiply by the reciprocal and simplify. In this case, we multiply both sides of the equation by -5/6.

-6 * (-5/6) = 5.

The answer must be positive, as two negatives divided equal a positive.

I hope this helps!

The solution of expression after divide is,

⇒ 5

We have to given that;

⇒ - 6 is divided by - 1.2

Now, We can simplify as,

⇒ - 6 is divided by - 1.2

It can be written as,

⇒ - 6 ÷ - 1.2

⇒ - 6 / - 1.2

⇒ 6 / 1.2

⇒ 60 / 12

⇒ 5

Thus, The solution of expression after divide is,

⇒ 5

Learn more about the divide visit:

https://brainly.com/question/28119824

#SPJ6

If g(x) = -4x + 5, find g(2x-1).

Answers

Answer:

- 8x + 9

Step-by-step explanation:

To evaluate g(2x - 1) substitute x = 2x - 1 into g(x), that is

g(2x - 1)

= - 4(2x - 1) + 5 ← distribute parenthesis and simplify

= - 8x + 4 + 5

= - 8x + 9

Answer:

= - 8x + 9

Step-by-step explanation:

= - 8x + 4 + 5

= - 8x + 9

Other Questions
A ramp with a mechanical advantage of 4 is used to move a 38-newton load. Whateffort force is needed to push the load up the ramp?A.9.5 NB.15.2NC.3.9 ND.0.11 N What is the area of this figure? Use 3.14 for pi.A.6.28 ft2B.18.28 ft2C.22.28 ft2D.28.56 ft2 Solve. Thank you so much! A picture is there. Which expression has a value of StartFraction 1 Over 36 EndFraction? (StartFraction 1 Over 108 EndFraction) cubed (StartFraction 1 Over 9 EndFraction) Superscript 4 (StartFraction 1 Over 6 EndFraction) squared (one-half) Superscript 5 SCIENCE: Scurvy can be cured if we consume1 pointcheese, egg yolkCitrus fruits, tomatoesnuts,legumesfish,dark leafy vegetables Which options provide correct details about the word fervently?Select all that apply.(at least two)A) An antonym of fervently is joyfully.B) An antonym of fervently is casually.C) A synonym of fervently is stealthily.D) A synonym of fervently is passionately. Cell phone Company ABC charges a $12 start-up fee plus $0.08 per minute, x/ Cell phone Company WYZ charges $0.20 per minute, x, with no start-up fee. Which function represents the difference in cost between Company ABC and Company WYZ? A 360. g iron rod is placed into 750.0 g of water at 22.5C. The water temperature rises to 46.7C. What was the initial temperature of the iron rod? If b = the number of blue bikes, which algebraic expression represents thephrase below?the sum of the number of blue bikes and the 9 red bikesA. b=9B. b +9C. b-9D. b.9SUBMI What is an example of Consumer Sovereignty in economics Tamera and Adelina are throwing a birthday party for their friend.Tamera invited 5 fewer friends than Adelina. Together they invited 47 guests.How many guests did each girl invite? pls say the 5 methods of preventing rusting of iron While the five _ pertain to tools for studying, the five _ pertain to whatgeographers will study A glass vase factory recently produced 3 vases that were chipped and 12 vases that were not. What is the experimental probability that the next vase will be chipped?Write your answer as a fraction or whole number. The allele for a widows peak has complete dominance over the allele for a straight hairline (w). What is the probability of two parents heterogeneous for a widows peak (Ww) having an offspring without a widows peak? Every society or group of people on Earth tends to define itself in part by:A.who it is not.B. its teas.c. fandom association.D. its alphabet. What makes things concurrent? The numerator of afraction is 1 more thantwice its denominator. If 4is added to both thenumerator and thedenominator, the fractionposinvereduces to 3. Find thedenominator. Shankar Company uses a perpetual system to record inventory transactions. The company purchases inventory on account on February 2 for $22,000. In addition to the cost of inventory, the company also pays $420 for freight charges associated with the purchase on the same day.Record the purchase of inventory on February 2, including the freight charges. (If no entry is required for a transaction/event, select "No journal entry required" in the first account field.)1. Record the purchase of inventory on account.2. Record the payment of freight charges in cash. A woman was found strangled in the basement of her home where she lived with her husband. The crime scene was processed and investigators left. Thenext day, the medical examiner reported that a scalloped, woven belt was used to strangle the victim. The police asked the husband to search the houseagain to look for such a belt and he agreed. A belt was found that matched marks on the victim. The man was arrested. His attorney made a motion that thebelt be excluded from the evidence as he contended that the search was not legal. Which statement is true?A)The belt should be excluded because the search was not legal. The police did not obtain a search warrant, as needed according to the FourthAmendment to the U.S. Constitution.B) The belt is admissible as evidence, according to Mincey v. Arizona.C)The belt should be excluded because the search was not legal. The police did not obtain a search warrant, as needed according to Michigan v. Tyler.D)The belt is admissible because the man gave his consent to the second search.